If a, b, and c are positive numbers, is a < b < c

This topic has expert replies
User avatar
Newbie | Next Rank: 10 Posts
Posts: 5
Joined: Fri Aug 05, 2011 9:45 am
If a, b, and c are positive numbers, is a < b < c

(1) ab = bc
(2) ac = bc

OA:D Explain.

User avatar
Master | Next Rank: 500 Posts
Posts: 134
Joined: Fri Apr 06, 2012 3:11 am
Thanked: 35 times
Followed by:5 members

by Shalabh's Quants » Fri Apr 13, 2012 9:50 pm
vinodhinir wrote:If a, b, and c are positive numbers, is a < b < c

(1) ab = bc
(2) ac = bc

OA:D Explain.
Stat. 1....

Since ab = bc, by cancelling b, we get a = c => It means a is not less than c.Ans is No.

Sufficient condn.

Stat. 2....

Since ac = bc, by cancelling c, we get a = b => It means a is not less than b.Ans is No.

Sufficient condn.
Shalabh Jain,
e-GMAT Instructor

User avatar
GMAT Instructor
Posts: 15539
Joined: Tue May 25, 2010 12:04 pm
Location: New York, NY
Thanked: 13060 times
Followed by:1906 members
GMAT Score:790

by GMATGuruNY » Fri Apr 13, 2012 10:00 pm
vinodhinir wrote:If a, b, and c are positive numbers, is a < b < c

(1) ab = bc
(2) ac = bc

OA:D Explain.
Statement 1: ab = bc
Since b>0, we can divide by b.
The result: a=c.
Thus, it is not true that a<c.
SUFFICIENT.

Statement 2: ac = bc
Since c>0, we can divide by c.
The result: a=b.
Thus, it is not true that a<b.
SUFFICIENT.

The correct answer is D.
Private tutor exclusively for the GMAT and GRE, with over 20 years of experience.
Followed here and elsewhere by over 1900 test-takers.
I have worked with students based in the US, Australia, Taiwan, China, Tajikistan, Kuwait, Saudi Arabia -- a long list of countries.
My students have been admitted to HBS, CBS, Tuck, Yale, Stern, Fuqua -- a long list of top programs.

As a tutor, I don't simply teach you how I would approach problems.
I unlock the best way for YOU to solve problems.

For more information, please email me (Mitch Hunt) at [email protected].
Student Review #1
Student Review #2
Student Review #3

User avatar
Newbie | Next Rank: 10 Posts
Posts: 5
Joined: Fri Aug 05, 2011 9:45 am

by vinodhinir » Sun Apr 15, 2012 8:06 pm
it is not true that a<c.
but how can we conclude that a is not less than b

lets say if b is 3 and a is 2 then a<b ...
if it s the other way round then a > b

how can this be a sufficient condn ?

I know am missing something very trivial here,but cannot think of what could it be ...

User avatar
GMAT Instructor
Posts: 15539
Joined: Tue May 25, 2010 12:04 pm
Location: New York, NY
Thanked: 13060 times
Followed by:1906 members
GMAT Score:790

by GMATGuruNY » Mon Apr 16, 2012 3:20 am
vinodhinir wrote:it is not true that a<c.
but how can we conclude that a is not less than b

lets say if b is 3 and a is 2 then a<b ...
if it s the other way round then a > b

how can this be a sufficient condn ?

I know am missing something very trivial here,but cannot think of what could it be ...
The information in statement 1 (a=c) renders the value of b irrelevant.
Since a=c, it is not possible that a<b<c.
SUFFICIENT.

There are only 3 possible scenarios:
If a=c and a<b, then c<b. Thus, a<b<c is not true.
If a=c and a=b, then a=b=c. Thus, a<b<c is not true.
If a=c and a>b, then c>b. Thus, a<b<c is not true.
In each case, it is not true that a<b<c.
SUFFICIENT.

Similar reasoning can be applied to statement 2.
Last edited by GMATGuruNY on Mon Apr 16, 2012 5:55 am, edited 1 time in total.
Private tutor exclusively for the GMAT and GRE, with over 20 years of experience.
Followed here and elsewhere by over 1900 test-takers.
I have worked with students based in the US, Australia, Taiwan, China, Tajikistan, Kuwait, Saudi Arabia -- a long list of countries.
My students have been admitted to HBS, CBS, Tuck, Yale, Stern, Fuqua -- a long list of top programs.

As a tutor, I don't simply teach you how I would approach problems.
I unlock the best way for YOU to solve problems.

For more information, please email me (Mitch Hunt) at [email protected].
Student Review #1
Student Review #2
Student Review #3

User avatar
Master | Next Rank: 500 Posts
Posts: 342
Joined: Wed Jul 08, 2009 8:50 am
Thanked: 214 times
Followed by:19 members
GMAT Score:740

by Birottam Dutta » Mon Apr 16, 2012 5:53 am
You have to understand that if you can prove any of a<b, a<c or b<c false, then it is sufficient to answer the question.

Remember in DS, the answer need not always be a correct one, even if the statements can prove the question to be incorrect, even that is considered as a sufficient statement.